2008 AIME II Problems/Problem 9

Revision as of 20:32, 3 April 2008 by Azjps (talk | contribs) (Solution 2: typo fix)

Problem

A particle is located on the coordinate plane at $(5,0)$. Define a move for the particle as a counterclockwise rotation of $\pi/4$ radians about the origin followed by a translation of $10$ units in the positive $x$-direction. Given that the particle's position after $150$ moves is $(p,q)$, find the greatest integer less than or equal to $|p| + |q|$.

Solution

Solution 1

Show periodic with $8$ steps, then invert twice. Template:Incomplete

Solution 2

Let the particle's position be represented by a complex number. The transformation takes $z$ to $f(z) = az + b$ where $a = e^{i\pi/4} = \frac {\sqrt {2}}{2} + i\frac {\sqrt {2}}{2}$ and $b = 10$. We let $a_0 = 5$ and $a_{n + 1} = f(a_n)$ so that we want to find $a_{150}$.

Basically, the thing comes out to \[a_{150} = (((5a + 10)a + 10)a + 10...) = 5a^{150} + 10 a^{149} + 10a^{149} ... + 10\] Notice that \[10(a^{150} + \ldots + 1) = 10(1 + a + \ldots + a^6) = - 10(a^7) = - 10( - \sqrt {2}/2 - i\sqrt {2}/2)\] Furthermore, $5a^{150} = - 5i$. So our final answer is \[5\sqrt {2} + 5(\sqrt {2} + 1) \approx 19.1 \Longrightarrow \boxed{019}.\]

See also

2008 AIME II (ProblemsAnswer KeyResources)
Preceded by
Problem 8
Followed by
Problem 10
1 2 3 4 5 6 7 8 9 10 11 12 13 14 15
All AIME Problems and Solutions